-
Notifications
You must be signed in to change notification settings - Fork 2
/
lista03.tex
518 lines (471 loc) · 25.2 KB
/
lista03.tex
1
2
3
4
5
6
7
8
9
10
11
12
13
14
15
16
17
18
19
20
21
22
23
24
25
26
27
28
29
30
31
32
33
34
35
36
37
38
39
40
41
42
43
44
45
46
47
48
49
50
51
52
53
54
55
56
57
58
59
60
61
62
63
64
65
66
67
68
69
70
71
72
73
74
75
76
77
78
79
80
81
82
83
84
85
86
87
88
89
90
91
92
93
94
95
96
97
98
99
100
101
102
103
104
105
106
107
108
109
110
111
112
113
114
115
116
117
118
119
120
121
122
123
124
125
126
127
128
129
130
131
132
133
134
135
136
137
138
139
140
141
142
143
144
145
146
147
148
149
150
151
152
153
154
155
156
157
158
159
160
161
162
163
164
165
166
167
168
169
170
171
172
173
174
175
176
177
178
179
180
181
182
183
184
185
186
187
188
189
190
191
192
193
194
195
196
197
198
199
200
201
202
203
204
205
206
207
208
209
210
211
212
213
214
215
216
217
218
219
220
221
222
223
224
225
226
227
228
229
230
231
232
233
234
235
236
237
238
239
240
241
242
243
244
245
246
247
248
249
250
251
252
253
254
255
256
257
258
259
260
261
262
263
264
265
266
267
268
269
270
271
272
273
274
275
276
277
278
279
280
281
282
283
284
285
286
287
288
289
290
291
292
293
294
295
296
297
298
299
300
301
302
303
304
305
306
307
308
309
310
311
312
313
314
315
316
317
318
319
320
321
322
323
324
325
326
327
328
329
330
331
332
333
334
335
336
337
338
339
340
341
342
343
344
345
346
347
348
349
350
351
352
353
354
355
356
357
358
359
360
361
362
363
364
365
366
367
368
369
370
371
372
373
374
375
376
377
378
379
380
381
382
383
384
385
386
387
388
389
390
391
392
393
394
395
396
397
398
399
400
401
402
403
404
405
406
407
408
409
410
411
412
413
414
415
416
417
418
419
420
421
422
423
424
425
426
427
428
429
430
431
432
433
434
435
436
437
438
439
440
441
442
443
444
445
446
447
448
449
450
451
452
453
454
455
456
457
458
459
460
461
462
463
464
465
466
467
468
469
470
471
472
473
474
475
476
477
478
479
480
481
482
483
484
485
486
487
488
489
490
491
492
493
494
495
496
497
498
499
500
501
502
503
504
505
506
507
508
509
510
511
512
513
514
515
516
517
518
% Filename: lista03.tex
%
% This code is part of 'Solutions for MT402, Matrizes'
%
% Description: This file corresponds to the solutions of homework sheet 03.
%
% Created: 11.03.12 08:00:00 AM
% Last Change: 29.06.12 05:34:31 PM
%
% Authors:
% - Raniere Silva (2012): initial version
%
% Copyright (c) 2012 Raniere Silva <[email protected]>
%
% This work is licensed under the Creative Commons Attribution-ShareAlike 3.0 Unported License. To view a copy of this license, visit http://creativecommons.org/licenses/by-sa/3.0/ or send a letter to Creative Commons, 444 Castro Street, Suite 900, Mountain View, California, 94041, USA.
%
% This work is distributed in the hope that it will be useful, but WITHOUT ANY WARRANTY; without even the implied warranty of MERCHANTABILITY or FITNESS FOR A PARTICULAR PURPOSE.
%
\begin{questions}
\section*{Matrizes com Estruturas de Blocos}
\question[Ver exemplo 3.7.3, p\'{a}gina 119, do Meyer\nocite{Meyer:2000:matrix}] Considere as matrizes n\~{a}o singulares: $A: r \times s$ e $B: s \times r$.
\begin{parts}
\part Demonstre (sem usar multiplica\c{c}\~{a}o direta), que se
\[
M = \begin{bmatrix}
A & 0 \\
0 & B
\end{bmatrix}
\]
ent\~{a}o,
\[
M^{-1} = \begin{bmatrix}
A^{-1} & 0 \\
0 & B^{-1}
\end{bmatrix}.
\]
\begin{solution}
\begin{align*}
\left[ \begin{array}[]{cc|cc}
A & 0 & I & 0 \\
0 & B & 0 & I
\end{array} \right] &\equiv \left[ \begin{array}[]{cc|cc}
I & 0 & A^{-1} & 0 \\
0 & B & 0 & I
\end{array} \right] \\
&\equiv \left[ \begin{array}[]{cc|cc}
I & 0 & A^{-1} & 0 \\
0 & I & 0 & B^{-1}
\end{array} \right].
\end{align*}
\end{solution}
\part Se
\[
M = \begin{bmatrix}
A & C \\
0 & B
\end{bmatrix}
\]
ent\~{a}o,
\[
M^{-1} = \begin{bmatrix}
A^{-1} & - A^{-1} C B^{-1} \\
0 & B^{-1}
\end{bmatrix}.
\]
\begin{solution}
\begin{align*}
\left[ \begin{array}[]{cc|cc}
A & C & I & 0 \\
0 & B & 0 & I
\end{array} \right] &\equiv \left[ \begin{array}[]{cc|cc}
I & A^{-1} C & A^{-1} & 0 \\
0 & B & 0 & I
\end{array} \right] \\
&\equiv \left[ \begin{array}[]{cc|cc}
I & A^{-1} C & A^{-1} & 0 \\
0 & I & 0 & B^{-1}
\end{array} \right] \\
&\equiv \left[ \begin{array}[]{cc|cc}
I & 0 & A^{-1} & -A^{-1} C B^{-1} \\
0 & I & 0 & B^{-1}
\end{array} \right].
\end{align*}
\end{solution}
\part Se
\[
M = \begin{bmatrix}
A & C \\
D & B
\end{bmatrix},
\]
$C: r \times s$ e $D: s \times r$, ent\~{a}o
\[
M ^{-1} = \begin{bmatrix}
A^{-1} + A^{-1} C S^{-1} D A^{-1} & -A^{-1} C S^{-1} \\
-S^{-1} D A^{-1} & S^{-1}
\end{bmatrix},
\]
onde $S = B - D A^{-1} C$ \'{e} o complemento de Schur de $A$. Observe que a condi\c{c}\~{a}o que $S$ tem que ser invers\'{i}vel surge no desenvolvimento do c\'{a}lculo da inversa de $M$.
\begin{solution}
\begin{align*}
\left[ \begin{array}[]{cc|cc}
A & C & I & 0 \\
D & B & 0 & I
\end{array} \right] &\equiv \left[ \begin{array}[]{cc|cc}
I & A^{-1} C & A^{-1} & 0 \\
D & B & 0 & I
\end{array} \right] \\
&\equiv \left[ \begin{array}[]{cc|cc}
I & A^{-1} C & A^{-1} & 0 \\
0 & B - D A^{-1} C & - D A^{-1} & I
\end{array} \right] \\
&\equiv \left[ \begin{array}[]{cc|cc}
I & A^{-1} C & A^{-1} & 0 \\
0 & S & - D A^{-1} & I
\end{array} \right] \\
&\equiv \left[ \begin{array}[]{cc|cc}
I & A^{-1} C & A^{-1} & 0 \\
0 & I & S^{-1} (- D A^{-1}) & S^{-1}
\end{array} \right] \\
&\equiv \left[ \begin{array}[]{cc|cc}
I & 0 & A^{-1} + (A^{-1} C) S^{-1} (D A^{-1}) & -A^{-1} C S^{-1} \\
0 & I & S^{-1} (- D A^{-1}) & S^{-1}
\end{array} \right].
\end{align*}
\end{solution}
\part Considere $M: (r + s) \times (r + s)$ dada por
\[
M = \begin{bmatrix}
A & C \\
D & B
\end{bmatrix},
\]
com $A: r \times r$ e $B: s \times s$. Verifique que esta matriz pode ser fatorada na forma $M = W Y$ onde
\[
W = \begin{bmatrix}
I_r & 0 \\
D A^{-1} & I_s
\end{bmatrix} \text{ e } Y = \begin{bmatrix}
A & C \\
0 & S
\end{bmatrix}.
\]
\begin{solution}
Para a verifica\c{c}\~{a}o utilizaremos a multiplica\c{c}\~{a}o direta:
\begin{align*}
M = W Y &= \begin{bmatrix}
I_r & 0 \\
D A^{-1} & I_s
\end{bmatrix} \begin{bmatrix}
A & C \\
0 & S
\end{bmatrix} \\
&= \begin{bmatrix}
A & C \\
D & D A^{-1} C + S
\end{bmatrix} \\
&= \begin{bmatrix}
A & C \\
D & D A^{-1} C + \left( B - D A^{-1} C \right)
\end{bmatrix} \\
&= \begin{bmatrix}
A & C \\
D & B
\end{bmatrix}.
\end{align*}
\end{solution}
\part[Ver equa\c{c}\~{a}o (6.1.16), p\'{a}gina 467, do Meyer\nocite{Meyer:2000:matrix}] Considerando a matriz $M$ do item anterior, demonstre que $\det (M) = \det (A) \det (S)$, onde $S = B - D A^{-1} C$ \'{e} o complemento de Schur de $A$.
\begin{solution}
% TODO Fazer esse exerc\'{i}cio.
\end{solution}
\part Repita os dois itens anteriores trabalhando com a forma fatorada de $M$, $M = W Y$, na qual um dos fatores depende do complemento de Schur de $B$, $T = A - C B^{-1} D$, obtendo assim o resultado:
\[
\det (M) = \det (B) \det (T) = \det (A) \det (S)
\]
\begin{solution}
Para a verifica\c{c}\~{a}o utilizaremos a multiplica\c{c}\~{a}o direta:
\begin{align*}
M = W Y &= \begin{bmatrix}
I_r & C B^{-1} \\
0 & I_s
\end{bmatrix} \begin{bmatrix}
T & 0 \\
D & B
\end{bmatrix} \\
&= \begin{bmatrix}
T + C B^{-1} C & C \\
D & B
\end{bmatrix} \\
&= \begin{bmatrix}
A - C B^{-1} D + C B^{-1} C & C \\
D & B
\end{bmatrix} \\
&= \begin{bmatrix}
A & C \\
D & B
\end{bmatrix}.
\end{align*}
% TODO: demonstrar que $\det (M) = \det (B) \det (T)$.
\end{solution}
\end{parts}
\section*{Subespa\c{c}os Fundamentais de A}
Os quatro espa\c{c}os fundamentais associados \`{a} matriz $A$ s\~{a}o:
\begin{itemize}
\item Espa\c{c}o coluna de $A$ ou espa\c{c}o imagem de $A$: $\EI (A) = \left\{ y \in \mathbb{R}^m \mid y = A x, x \in \mathbb{R}^n \right\}$,
\item N\'{u}cleo de $A$ ou espa\c{c}o nulo de $A$: $\EN (A) = \left\{ x \in \mathbb{R}^n \mid A x = 0 \right\}$,
\item Espa\c{c}o linha de $A$ ou espa\c{c}o coluna de $A^t$: $\EI (A^t) = \left\{ x \in \mathbb{R}^n \mid x = A^t y , y \in \mathbb{R}^m \right\}$,
\item N\'{u}cleo de $A^t$ ou espa\c{c}o nulo de $A^t$: $\EN (A^t) = \left\{ y \in \mathbb{R}^m \mid A^t y = 0 \right\}$.
\end{itemize}
\question Demonstre que o sistema linear $A x = b$ admite solu\c{c}\~{a}o se e somente se $\posto ([A \ b]) = \posto (A)$.
\begin{solution}
Primeiro vamos demonstrar que se o sistema linear $A x = b$ admite solu\c{c}\~{a}o ent\~{a}o $\posto ([A \ b]) = \posto (A)$.
Se o sistema linear $A x = b$ admite solu\c{c}\~{a}o existe $x^*$ tal que $b = \sum_j A_{\mdot j} x^*_j$, i.e., $b$ \'{e} uma combina\c{c}\~{a}o linear de $A$. Portanto, $\posto ([A \ b]) = \posto (A)$.
Agora vamos demonstrar que o $\posto ([A \ b]) = \posto (A)$ ent\~{a}o o sistema linear $A x = b$ admite solu\c{c}\~{a}o.
Se $\posto ([A \ b]) = \posto (A)$ o conjunto das colunas de $[A \ b]$ \'{e} linearmente dependente, i.e., existe $(\alpha, \beta)$ tal que
\begin{align*}
\left( \sum_j \alpha_j A_{\mdot j} + \beta b \right) &= 0 \\
\sum_j - \frac{\alpha_j}{\beta} A_{\mdot j} &= b.
\end{align*}
Logo, o sistema linear $A x = b$ admite solu\c{c}\~{a}o.
\end{solution}
\question Considere $x^*$ uma solu\c{c}\~{a}o de $A x = b$ e $S$ o conjunto das solu\c{c}\~{o}es de $A x = b$. Demonstre que $S = \left\{ x^* + z, \forall z \in \EN (A) \right\}$.
\begin{solution}
Seja $\bar{x} \in \left\{ x^* + z, \forall z \in \EN (A) \right\}$, ent\~{a}o
\begin{align*}
A \bar{x} &= A \left( x^* + z \right) && \bar{x} \in \left\{ x^* + z, \forall z \in \EN (A) \right\} \\
&= A x^* + A z \\
&= b + 0 && A x^* = b \text{ e } z \in \EN (A) \\
&= b.
\end{align*}
\end{solution}
\question Demonstre que a solu\c{c}\~{a}o do sistema linear \'{e} \'{u}nica se e somente se $\EN (A) = \left\{ 0 \right\}$.
\begin{solution}
Primeiro vamos demonstrar que se a solu\c{c}\~{a}o do sistema linear \'{e} \'{u}nica ent\~{a}o $\EN (A) = \left\{ 0 \right\}$.
Seja $x^*$ a solu\c{c}\~{a}o \'{u}nica do sistema linear $A x = b$ e $x = x^* + y$ qualquer ponto do dom\'{i}nio. Ent\~{a}o
\begin{align*}
A x &= A \left( x^* + y \right) \\
&= A x^* + A y \\
&= b + A y.
\end{align*}
Logo, se existir $y \neq 0$ tal que $A y = 0$ o sistema linear teria mais de uma solu\c{c}\~{a}o. Como o sistema linear admite uma \'{u}nica solu\c{c}\~{a}o concluimos que $\EN (A) = \left\{ 0 \right\}$.
Agora vamos demonstrar que se $\EN (A) = \left\{ 0 \right\}$ a solu\c{c}\~{a}o do sistema linear \'{e} \'{u}nica.
Como demonstrado anteriormente, o conjunto das solu\c{c}\~{o}es do sistema linear $A x = b$, denotado por $S$, \'{e} dado por $S = \left\{ x^* + z, \forall z \in \EN (A) \right\}$, onde $x^*$ \'{e} uma solu\c{c}\~{a}o do sistema linear. Se $\EN (A) = \left\{ 0 \right\}$ notamos que $S = \left\{ x^* \right\}$ e portanto a solu\c{c}\~{a}o dos sistema linear \'{e} \'{u}nica.
\end{solution}
\question Para todo $b \in \mathbb{R}^m$, o sistema linear $A x = b$, $A : m \times n$, admite uma \'{u}nica solu\c{c}\~{a}o se e somente se $m = n$ e $\posto (A) = n$.
\begin{solution}
Primeiro vamos mostrar que se o sistema linear $A x = b$, $A: m \times n$, admite uma \'{u}nica solu\c{c}\~{a}o para todo $b \in \mathbb{R}^m$ ent\~{a}o $m = n$ e $\posto (A) = n$.
Como o sistema linear admite solu\c{c}\~{a}o para todo $b \in \mathbb{R}^m$ temos que $A$ possue exatamente $m$ linhas ($m = n$) e no m\'{i}nimo $m$ colunas, sendo $m$ colunas linearmente independentes ($\posto (A) = n$). E como o sistema linear admite uma \'{u}nica solu\c{c}\~{a}o ent\~{a}o $A$ possue e$b$ \'{e} uma combina\c{c}\~{a}o linear das colunas de $A$ e as colunas de $A$ s\~{a}o linearmente independentes.
Agora vamos mostrar que se $m = n$ e $\posto (A) = n$ ent\~{a}o o sistema linear $A x = b$, $A: m \times n$ admite uma \'{u}nica solu\c{c}\~{a}o.
\end{solution}
\question Demonstre (sem usar o Teorema do N\'{u}cleo e Imagem) que $\EN (A) = \left\{ 0 \right\}$ se e somente se $A$ tem as $n$ colunas linearmente independentes.
\begin{solution}
Primeiro vamos mostrar que se $\EN (A) = \left\{ 0 \right\}$ ent\~{a}o $A$ tem as $n$ colunas linearmente independentes.
Se $\EN (A) = \left\{ 0 \right\}$ temos, como demonstrado anteriormente, que o sistema linear $A x = b$ admite uma \'{u}nica solu\c{c}\~{a}o e isso ocorre somente se as $n$ colunas de $A$ forem linearmente independentes.
Agora vamos mostrar que se $A$ tem as $n$ colunas linearmente independentes ent\~{a}o $\EN (A) = \left\{ 0 \right\}$.
Se $A$ tem as $n$ colunas linearmente independentes ent\~{a}o
\[
\sum_j \alpha_j A_{\mdot j} = 0 \leftrightarrow \alpha_j = 0, \ \forall j.
\]
Logo, $\EN (A) = \left\{ 0 \right\}$.
\end{solution}
\question Julgue verdadeiro ou falso a afirma\c{c}\~{a}o abaixo:
\begin{quote}
Se $A: m \times n$ para $n > m$ ent\~{a}o $\EN (A) \neq \left\{ 0 \right\}$.
\end{quote}
\begin{solution}
A afirma\c{c}\~{a}o \'{e} verdadeira pois se $A: m \times n$ para $n > m$ temos que pelo menos $\left( n - m \right)$ colunas de $A$ que s\~{a}o linearmente dependentes, i.e.,
\[
\exists \alpha_j \neq 0 \rightarrow \sum_j \alpha_j A_{\mdot j} = 0.
\]
Logo, $\EN (A) \neq \left\{ 0 \right\}$.
\end{solution}
\question Analisando os resultados demonstrados nos itens anteriores, fa\c{c}a um resumo colocando as condi\c{c}\~{o}es que a matriz $A: m \times n$ deve satisfazer para garatir a exist\^{e}ncia e unicidade da solu\c{c}\~{a}o de $A x = b$, para qualquer $b \in \mathbb{R}^m$.
\begin{solution}
Para maiores detalhes ver resumo na p\'{a}gina 70 do Meyer\nocite{Meyer:2000:matrix}.
\begin{quote}
O sistema linear possue solu\c{c}\~{a}o única se e somente se alguma das condi\c{c}\~{o}es abaixo for verdadeira:
\begin{enumerate}
\item $\text{posto}(A) = n$ que \'{e} igual ao número de vari\'{a}veis desconhecidas;
\item n\~{a}o existem vari\'{a}veis livres;
\item o sistema homogeneo associado possue apenas a solu\c{c}\~{a}o trivial.
\end{enumerate}
\end{quote}
\end{solution}
\question Considere o sistema linear $A x = b$ onde
\[
A = \begin{bmatrix}
2 & 5 \\
4 & 10
\end{bmatrix} \text{ e } b = \begin{bmatrix}
3 \\
6
\end{bmatrix}.
\]
Encontre o conjunto de solu\c{c}\~{o}es para este sistema linear e interprete geometricamente o resultado abaixo:
\begin{quote}
Conhecida uma solu\c{c}\~{a}o $x^*$ qualquer vetor da forma $x^* + w$ \'{e} solu\c{c}\~{a}o para o sistema linear onde $w$ \'{e} um vetor qualquer do $\EN (A)$.
\end{quote}
\begin{solution}
O conjunto solu\c{c}\~{a}o do sistema linear, denotado por $S$, \'{e} dado por
\[
S = \left\{ (x, y) \in \mathbb{R}^2 \mid x = \left( 3 - 5 y \right) / 2 \right\}.
\]
\begin{center}
\begin{tikzpicture}[scale=0.5]
% Eixos
\draw[color=gray!70] (0,0) grid (12,12);
\draw[->] (-0.5,0) -- (12.5,0) node[below] {$x$};
\draw[->] (0,-0.5) -- (0,12.5) node[left] {$y$};
% Colunas de A
\draw[->] (0,0) -- (2,4) node[below right] {$(2,4)$};
\draw[->] (0,0) -- (5,10) node[below right] {$(5,10)$};
% Vetor B
\draw[->] (0,0) -- (3,6) node[below right] {$(3,6)$};
\end{tikzpicture}
\end{center}
\end{solution}
\question Demonstre que $\EN (A^t)$ e $\EI (A)$ s\~{a}o complementos ortogonais em $\mathbb{R}^m$. E que $\EN (A)$ e $\EI (A^t)$ s\~{a}o complementos ortogonais em $\mathbb{R}^n$.
\begin{solution}
Seja $y \in \EI (A)$ e $z \in \EN (A^t)$, i.e., existe $x$ tal que $A x = y$ e $A^t z = 0$. Ent\~{a}o
\begin{align*}
A x &= y \\
\left( A x \right)^t &= y^t \\
x^t A^t &= y^t \\
x^t A^t z &= y^t z \\
x^t 0 &= y^t z && z \in \EN (A^t) \\
0 &= y^t z.
\end{align*}
Seja $y \in \EI (A^t)$ e $z \in \EN(A)$, i.e., existe $x$ tal que $A^t x = y$ e $A z = 0$. Ent\~{a}o
\begin{align*}
A^t x &= y \\
\left( A^t x \right)^t &= y^t \\
x^t A &= y^t \\
x^t A z &= y^t z \\
x^t 0 &= y^t z && z \in \EN (A) \\
0 &= y^t z.
\end{align*}
\end{solution}
\section*{Resultados para posto de matriz}
\question[Exemplo 4.4.8, p\'{a}gina 206, do Meyer\nocite{Meyer:2000:matrix}] Demonstre que $\posto (A + B) \leq \posto (A) + \posto (B)$.
\begin{solution}
Note que
\begin{align*}
\EI(A + B) \subseteq \EI(A) + \EI(B)
\end{align*}
pois se $b \in \EI(A + B)$ ent\~{a}o existe $x$ tal que
\begin{align*}
b = (A + B) x = Ax + Bx \in \EI(A) + \EI(B).
\end{align*}
Mas se $M$ e $N$ s\~{a}o espa\c{c}os vetoriais tal que $M \subseteq N$ ent\~{a}o $\dim M \leq \dim N$. Utilizando isso com a f\'{o}rmula para a dimens\~{a}o da soma temos
\begin{align*}
\posto (A + B) &= \dim \EI(A + B) \\
&\leq \dim \left( \EI(A) + \EI(B) \right) \\
&= \dim \EI(A) + \dim \EI(B_ - \dim \left( \EI(A) \cap \EI(B) \right) \\
&\leq \dim \EI(A) + \dim \EI(B) \\
&= \posto (A) + \posto (B).
\end{align*}
\end{solution}
\question Se $A: m \times p$ e $B: p \times n$, demonstre que:
\begin{parts}
\part[Equa\c{c}\~{a}o (4.5.1), p\'{a}gina 210, do Meyer\nocite{Meyer:2000:matrix}] $\posto (A B) = \posto (B) - \dim (\EN (A) \cap \EI (B))$.
\begin{solution}
Come\c{c}ando com uma base $S = \left\{ x_1, x_2, \ldots, x_x \right\}$ para $\EN(A) \cap \EI(B)$, notamos que $\EN(A) \cap \EI(B) \subseteq \EI(B)$. Se $\dim \EI(B) = s + t$, ent\~{a}o existe um conjunto $S_\text{ext} = \left\{ z_1, z_2, \ldots, z_t \right\}$ tal que $B = \left\{ x_1, \ldots, x_s, z_1, \ldots, z_t \right\}$ \'{e} base para $\EN(B)$. Nosso objetivo \'{e} provar que $\dim \EI(AB) = t$ e isso \'{e} feito mostrando $T = \left\{ A z_1, A z_2, \ldots, A z_t \right\}$ \'{e} base para $\EI(AB)$. $T$ gera $\EI(AB)$ porque se $b \in \EI(AB)$ ent\~{a}o $b = A B y$ para algum $y$, mas $B y \in \EI(B)$ implica em $B y = \sum_{i = 1}^s \nu_i z_i$, e portanto
\begin{align*}
b = A \left( \sum_{i = 1}^s \varepsilon_i x_i + \sum_{i = 1}^t \nu_i z_i \right) = \sum_{i = 1}^s \varepsilon_i A x_i + \sum_{i = 1}^t \nu_i A z_i = \sum_{i = 1}^t \nu_i A z_i.
\end{align*}
$T$ \'{e} linearmente independente porque se $0 = \sum_{i = 1}^t \alpha_i A z_i = A \sum_{u = 1}^t \alpha_i x_i$ ent\~{a}o $\sum_{i = 1}^t \alpha_i z_i \in \EN(A) \cap \EI(B)$. Portanto existe escalares $\beta_j$ tal que
\begin{align*}
\sum_{i = 1}^t \alpha_i z_i = \sum_{j = 1}^s \beta_j x_j
\end{align*}
e a única solu\c{c}\~{a}o para $\alpha_i$ e $\beta_j$ \'{e} a solu\c{c}\~{a}o trivial porque $B$ \'{e} um conjunto independente. Logo $T$ \'{e} base para $\EI(AB)$ e $t = \dim \EI(AB) = \posto (AB)$ e
\begin{align*}
\posto (B) = \dim \EI(B) = s + t = \dim \EN(A) \cap \EI(B) + \posto (AB).
\end{align*}
\end{solution}
\part[Equa\c{c}\~{o}es (4.5.2) e (4.5.3), p\'{a}gina 211, do Meyer\nocite{Meyer:2000:matrix}] $\posto (A) + \posto (B) - n \leq \posto (A B) \leq \min \left\{ \posto (A), \posto (B) \right\}$.
\begin{solution}
Primeiro vamos mostrar que $\posto (AB) \leq \min \left\{ \posto(A), \posto(B) \right\}$.
Escrevendo
\begin{align*}
\posto(AB) = \posto(B) - \dim \EN(A) \cap \EI(B) \leq \posto(B)
\end{align*}
e
\begin{align*}
\posto(AB) = \posto(AB)^t = \posto(B^t A^b) \leq \posto(A^t) = \posto(A).
\end{align*}
Logo,
\begin{align*}
\posto(A) \leq min \left\{ \posto(A), \posto(B) \right\}.
\end{align*}
Agora vamos mostrar que $\posto(A) + \posto(B) - n \leq \posto(AB)$.
Notamos que $\EN(A) \cap \EI(B) \subseteq \EN(A)$ e que se $M$ e $N$ s\~{a}o espa\c{c}os vetoriais tal que $M \subseteq N$ ent\~{a}o $\dim M \leq \dim N$. Portanto,
\begin{align*}
\dim \EN(A) \cap \EI(B) \leq \dim \EN(A) = n - \posto (A)
\end{align*}
e utilizando o limitante inferior para $\posto(AB)$ encontrado anteriormente temos
\begin{align*}
\posto(AB) = \posto(B) = \dim \EN(A) \cap \EI(B) \geq \posto(B) + \posto(A) - n.
\end{align*}
\end{solution}
\end{parts}
\question Para matrizes $A$ e $B$ demonstre que $\EI (A B) \subseteq \EI (A)$ e $\EN (B) \subseteq \EN (A B)$.
\begin{solution}
Pela defini\c{c}\~{a}o de espa\c{c}o imagem temos que
\begin{align*}
\EI (A B) &= \left\{ y \mid A B x = y \right\}, \\
\EI (A) &= \left\{ z \mid A w = z \right\}.
\end{align*}
Notamos ent\~{a}o que $\EI (A B) \subseteq \EI (A)$ pois \'{e} poss\'{i}vel que exista $w$ que n\~{a}o perten\c{c}a a imagem de $B$.
Pela defini\c{c}\~{a}o de espa\c{c}o nulo temos que
\begin{align*}
\EN (A B) &= \left\{ x \mid A B x = 0 \right\}, \\
\EN (B) &= \left\{ y \mid B y = 0 \right\}.
\end{align*}
Notamos ent\~{a}o que $\EN (B) \subseteq \EN (A B)$ pois para todo $y \in \EN (B)$ verifica-se $A B y = 0$ mas pode existir $x$ tal que $B x \neq 0$ mas $ A B x = 0$.
\end{solution}
\question Demonstre que:
\begin{parts}
\part[Equa\c{c}\~{a}o (4.5.4), p\'{a}gina 212, do Meyer\nocite{Meyer:2000:matrix}] $\posto (A^t A) = \posto (A) = \posto (A A^t)$.
\begin{solution}
Note que $\EN(A^t) \cap \EI(A) = \left\{ 0 \right\}$ pois
\begin{align*}
x \in \EN(A^t) \cap \EI(R) &\implies A^t x = 0, x = A y \\
&\implies x^t x = y^t A^t x = 0 \\
&\implies \sum x_i^2 = 0 \\
&\implies x = 0.
\end{align*}
Logo,
\begin{align*}
\posto(A^t A) = \posto(A) - \dim \EN(A^t) \cap \EI(A) = \posto(A).
\end{align*}
Permutando a ordem de $A$ e $A^t$ temos
\begin{align*}
\posto(A A^t) = \posto(A^t) - \dim \EN(A) \cap \EI(A^t) = \posto(A^t).
\end{align*}
\end{solution}
\part[Equa\c{c}\~{a}o (4.5.5), p\'{a}gina 212, do Meyer\nocite{Meyer:2000:matrix}] $\EI (A^t A) = \EI (A^t)$ e $\EI (A A^t) = \EI (A)$.
\begin{solution}
Note que $\EI(AB) \subseteq \EI(A)$ e portanto $\EI(A^t A) \subset \EI(A^t)$. Logo
\begin{align*}
\dim \EI(A^t A) = \posto(A^t A) = \posto(A) = \posto(A^t) = \dim \EN(A^t)
\end{align*}
e permutando a ordem de $A$ e $A^t$ temos
\begin{align*}
\dim \EI(A A^t) = \posto(A A^t) = \posto(A^t) = \posto(A) = \dim \EN(A).
\end{align*}
\end{solution}
\part[Equa\c{c}\~{a}o (4.5.6), p\'{a}gina 212, do Meyer\nocite{Meyer:2000:matrix}] $\EN (A^t A) = \EN (A)$ e $\EN (A A^t) = \EN (A^t)$.
\begin{solution}
Note que $\EN(B) \subset \EN(AB)$ e portanto $\EN(A) \subset \EN(A^t A)$. Logo,
\begin{align*}
\dim \EN(A) = n - \posto(A) = n - \posto(A^t A) = \dim \EN(A^t A)
\end{align*}
e permutando a ordem de $A$ e $A^t$ temos
\begin{align*}
\dim \EN(A^t) = n - \posto(A^t) = n - \posto(A A^t) = \dim \EN(A A^t).
\end{align*}
\end{solution}
\end{parts}
\question[Ver resumo, p\'{a}gina 214, do Meyer\nocite{Meyer:2000:matrix}] Considere o sistema linear $A x = b$, com $A : m \times n$ e o sistema $A^t A x = A^t b$ (sistema de equa\c{c}\~{o}es normais). Verifique se as informa\c{c}\~{o}es abaixo s\~{a}o verdadeiras ou falsas.
\begin{parts}
\part O sistema $A^t A x = A^t b$ sempre admite solu\c{c}\~{a}o, ainda que $A x = b$ n\~{a}o tenha solu\c{c}\~{a}o.
\begin{solution}
Se $A$ tiver posto completo ent\~{a}o a afirma\c{c}\~{a}o \'{e} verdadeira. Se $A$ n\~{a}o tiver posto completo ent\~{a}o a matriz $A^t A$ \'{e} singular e portanto o sistema $A^t A x = A^t b$ n\~{a}o admite solu\c{c}\~{a}o.
\end{solution}
\part Se os sitema linear $A x = b$ admite solu\c{c}\~{a}o ent\~{a}o os dois sistemas possuem o mesmo conjunto solu\c{c}\~{a}o.
\begin{solution}
Se o sistema linear $A x = b$ admite solu\c{c}\~{a}o \'{u}nica ent\~{a}o $A$ tem posto completo e portanto ambos os sistemas possuem o mesmo conjunto solu\c{c}\~{a}o. Se o sistema $A x = b$ possue infintas solu\c{c}\~{o}es ent\~{a}o $A$ n\~{a}o possue posto completo e portanto $A^t A$ \'{e} singular de forma que o sistema $A^t A x = A^t b$ n\~{a}o admite solu\c{c}\~{a}o.
\end{solution}
\end{parts}
\end{questions}